Difference between revisions of "2008 iTest Problems/Problem 18"

(Added modular arithmetic solution)
m
 
Line 6: Line 6:
  
 
Solve for <math>y</math> to get
 
Solve for <math>y</math> to get
<cmath>y = \frac{1909-19x}{20}</cmath>
+
<cmath>y = \frac{1909-19x}{20}.</cmath>
 
In order for <math>y</math> to be an positive integer, <math>1909-19x</math> must be a multiple of 20 greater than <math>0</math>, so <math>x \le 100</math>.  This means that the ones digit of <math>1909-19x</math> is <math>0</math> and the tens digit of <math>1909-19x</math> is even.   
 
In order for <math>y</math> to be an positive integer, <math>1909-19x</math> must be a multiple of 20 greater than <math>0</math>, so <math>x \le 100</math>.  This means that the ones digit of <math>1909-19x</math> is <math>0</math> and the tens digit of <math>1909-19x</math> is even.   
  

Latest revision as of 09:29, 23 June 2022

Problem

Find the number of lattice points that the line $19x+20y = 1909$ passes through in Quadrant I.

Solution 1

Solve for $y$ to get \[y = \frac{1909-19x}{20}.\] In order for $y$ to be an positive integer, $1909-19x$ must be a multiple of 20 greater than $0$, so $x \le 100$. This means that the ones digit of $1909-19x$ is $0$ and the tens digit of $1909-19x$ is even.

The ones digit of $1909-19x$ is $0$ when the last digit of $x$ is $1$, so the available options are $1, 11, 21 \cdots 91$. However, since $1909-19x=1909-20x+x$, the tens digit must be odd. Thus, the only values that work are $11$, $31$, $51$, $71$, and $91$, so there are only $\boxed{5}$ lattice points in the first quadrant.

Solution 2 (Modular Arithmetic)

As in Solution 1, we rearrange the equation to get \[y = \frac{1909 - 19x}{20}.\] This means $1909 - 19x$ must be positive and divisible by 20, and we know $x$ is an integer, so we set it congruent to $0 \pmod{20}$ and simplify from there: \[\begin{array}{rll} 1909 - 19x \equiv & 0  & \pmod{20} \\ 9 + x \equiv      & 0  & \pmod{20} \\ x \equiv          & -9 & \pmod{20} \\ x \equiv          & 11 & \pmod{20}. \end{array}\] Since our lattice points are in the first quadrant, $x$ is positive, so we can start listing off our solutions: $x = 11, 31, 51, 71, 91, 111...$. Noticing that $19(111) \approx 19(101) = 1919 > 1909$, we conclude that $111$ is too large, and so our solutions are $11, 31, 51, 71,$ and $91$, for a total of $\boxed{5}$ lattice points.

See Also

2008 iTest (Problems)
Preceded by:
Problem 17
Followed by:
Problem 19
1 2 3 4 5 6 7 8 9 10 11 12 13 14 15 16 17 18 19 20 21 22 23 24 25 26 27 28 29 30 31 32 33 34 35 36 37 38 39 40 41 42 43 44 45 46 47 48 49 50 51 52 53 54 55 56 57 58 59 60 61 62 63 64 65 66 67 68 69 70 71 72 73 74 75 76 77 78 79 80 81 82 83 84 85 86 87 88 89 90 91 92 93 94 95 96 97 98 99 100